Finding probability given joint pdf.

Click For Summary
To evaluate the probabilities P(X1 < X2|X1 < 2X2) and P(X1 < X2 < X3|X3 < 1), one must understand the implications of conditional probability. The condition X1 < 2X2 implies a specific probability density function that must be integrated to find the desired probabilities. The discussion highlights a potential misunderstanding regarding the notation, clarifying that the probabilities are conditioned on specific events. The approach involves calculating the joint probability densities under the given conditions. Understanding these concepts is essential for solving the problems accurately.
cookiesyum
Messages
72
Reaction score
0

Homework Statement



Let X1 X2 X3 be independent and identically distributed random variables with common pdf f(x) = e^-x 0<x<infinity, zero elsewhere.

Evaluate P(X1 < X2|X1 < 2X2) and P(X1 < X2 < X3|X3 < 1).


The Attempt at a Solution



Would appreciate any hints as to where to start...
 
Physics news on Phys.org
I haven't actually studied any statistics so there might be an easier way of doing this but this is how I'd do it...

Having a conditional probability means that you essentially have a 100% probability for the condition, that is

\int_{x_1 &lt; 2x_2} dx_1 dx_2 f(x_1, x_2) = 1

This gives you the probability density given the condition that x_1 < 2 x_2. You can then use that to find the probability that x_1 < x_2.
 
clamtrox said:
I haven't actually studied any statistics so there might be an easier way of doing this but this is how I'd do it...

Having a conditional probability means that you essentially have a 100% probability for the condition, that is

\int_{x_1 &lt; 2x_2} dx_1 dx_2 f(x_1, x_2) = 1

This gives you the probability density given the condition that x_1 < 2 x_2. You can then use that to find the probability that x_1 < x_2.

But I don't think the problem is saying X1 < 2X2 I think the phrasing was X2|X1 < 2X2. Am I thinking about it wrong?
 
The first problem reads out as "probability that x_1 < x_2, given that you already know that x_1 < 2x_2". The 2nd problem is "probability that x_1 < x_2 < x_3, given that you know that x_3 < 1". Atleast that is what this notation means as far as I know. :-)
 
Question: A clock's minute hand has length 4 and its hour hand has length 3. What is the distance between the tips at the moment when it is increasing most rapidly?(Putnam Exam Question) Answer: Making assumption that both the hands moves at constant angular velocities, the answer is ## \sqrt{7} .## But don't you think this assumption is somewhat doubtful and wrong?

Similar threads

  • · Replies 8 ·
Replies
8
Views
4K
Replies
11
Views
2K
  • · Replies 8 ·
Replies
8
Views
2K
Replies
1
Views
3K
Replies
7
Views
3K
  • · Replies 4 ·
Replies
4
Views
2K
  • · Replies 4 ·
Replies
4
Views
2K
  • · Replies 3 ·
Replies
3
Views
2K
  • · Replies 2 ·
Replies
2
Views
2K
  • · Replies 2 ·
Replies
2
Views
6K